LSAT and Law School Admissions Forum

Get expert LSAT preparation and law school admissions advice from PowerScore Test Preparation.

 Kdup
  • Posts: 31
  • Joined: Aug 14, 2017
|
#40947
Hi Powerscore,

For this question, I am having trouble understanding why "A" is the correct answer. I chose C. I thought that if everything was equal preceding the study and then one group was giving something that altered performance, wouldn't that strengthen the argument even more?
 James Finch
PowerScore Staff
  • PowerScore Staff
  • Posts: 943
  • Joined: Sep 06, 2017
|
#40973
Hi K,

As a Strengthen question about an experiment, we need to make sure that the proper experimental parameters are covered--the correct answer will usually give additional information that makes the experiment more likely to be valid. In this case, Plant A is our experimental group and Plant B is our control group, while the study is testing the effects of nutritious breakfasts on productivity (Plant A eats nutritious breakfasts for a month). However, we're not told what Plant B is given for breakfast, if anything. We are given the information that Plant A's productivity goes up, while Plant B's remains the same, so we know the change in productivity.

So what essential information are we missing to judge the validity of the study? WE don't know anything about the breakfasts that either plant was eating before the study, nor what Plant B was eating during the study. Without that information, we cannot judge whether the nutritious breakfast was the cause in the productivity change or not.

Answer choice (A) helps to make the conclusion more likely to be true, if not by much. If all the workers in Plant B had been already eating nutritious breakfasts before and during the study, for example, we wouldn't expect their productivity to change. It doesn't logically prove the conclusion, but this is a Strengthen, not Justify question, so it only has to make it more likely to be true to be the correct answer.

(C) doesn't have any effect on the conclusion, because we already know that Plant A's productivity went up, while Plant B's stayed the same. The baseline productivity levels are irrelevant. The issue we have is controlling for the breakfasts eaten at both plants before and during the study, information (C) doesn't provide.

Hope this clears things up!
 grunerlokka
  • Posts: 22
  • Joined: Jul 07, 2020
|
#85098
I am having trouble eliminating answers B and D here. B would assure that one key variable (since productivity is output per standard unit of time) was indeed constant for the two plants. D would strengthen that the lunches had a positive effect for plant A group, because they were more productive than plant B group despite having less vacation time than B. Like correct answer choice A, both B & D would be very weak strengtheners, but strengtheners nonetheless. What am I missing?
 Jeremy Press
PowerScore Staff
  • PowerScore Staff
  • Posts: 1000
  • Joined: Jun 12, 2017
|
#85113
Hi gruner,

I'm not sure I agree with you that answer choice B assures the productivity variable you describe is constant between the two groups. After all, answer choice B only mentions what time the two groups start working. It doesn't mention how long they worked in total or how much they produced. Without knowing more about how starting time relates to overall productivity in the two groups, answer choice B doesn't actually do anything for me. Even if you like the slight bit of consistency answer choice B describes across the two groups, it's doing very, very little to help the argument, and certainly not as much as answer choice A. Since the question stem wants you to pick the answer that "most strengthens" the argument, that can't be answer choice B.

The problem with answer choice D is that the per capita vacation days among the two groups for the duration of the study aren't revealing very much about what caused the workers who were at work to be productive. Maybe you had several workers at Plant B who took the whole month of the study off, increasing the per capita vacation time of the B group but not affecting the productivity of the workers who actually showed up to work. So, it's not a granular enough answer to help shed light on the issues here. Further, it has the same problem as answer choice B. Even if you spin a scenario where basically every worker at Plant A had less vacation time than every worker at Plant B, answer choice A is a much stronger answer for the reason James identifies: answer choice A fills a massive hole in the argument, the question of whether Plant B is a true "control group" when it comes to nutritious breakfasts.

I hope this helps!

Get the most out of your LSAT Prep Plus subscription.

Analyze and track your performance with our Testing and Analytics Package.